[试题] 104-2 马小康 流体力学 第二次期中考

楼主: sunhuang (茵茵)   2016-05-13 19:08:59
课程名称︰流体力学
课程性质︰机械系大二必修
课程教师︰马小康
开课学院:工学院
开课系所︰机械系
考试日期(年月日)︰105/5/13
考试时限(分钟):100 min
试题 :
(15%) (1) Underground water is to be pumped by a 80% efficiency 5-kW submerged
pump to a pool whose free surface is 30m above the underground water
level. The diameter of the pipe is 7cm on the intake side and 5cm on
the discharge side. Determine (a) the maximum flow rate of water and
(b) the pressure difference across the pump. Assume the elevation
difference between the pump inlet and ourlet and the effect of the
kinetic energy correction factors to be negligible. The density of
water is 1kg/L = 1000kg/m^3.
http://i.imgur.com/WUZ4TGB.jpg
(15%) (2) Please write (a) Bernoulli equation in the direction r normal to
streamlines for steady, incompressible flow. Where r is the local
radius of curvature, and r1 = λ- R and r2 = λ+ R. If velocity is
V = C/r, please (b) find the constant C.
http://i.imgur.com/fqBbxKL.jpg
(15%) (3) A periodic Karman vortex street is formed when a uniform stream
flows over a circular cylinder. Use the method of repeating
variables to generate a dimensionless relationship for Karman vortex
shedding frequency fk as a function of free stream speed V, fluid
density, and cylinder diameter D. fk = f(V,ρ,μ,D) n=5, Please show
all your work.
(15%) (4) For the following flow system with 1.5-cm-diameter,the gage pressure
at the inlet system is 200kPa during a shower and the toilet
reservior is full, please determine (a) the total hL and (b) the
minor losses coefficients KL between state 1 and 2 and between state
1 and 3.
http://i.imgur.com/F3ERipC.jpg
(20%) (5) Water accelerated by a nozzle to 35m/s strikes the vertical back
surface of a cart moving horizontally at a constant velocity 10m/s
in the flow direction. The mass flow rate of water through the
stationary nozzle is 30kg/s. After the strike, the water stream
splatters off in all directions in the plane of the back surface.
(a) Determine the force that needs to be applied by the brakes of
the cart to prevent it from accelerating. (b) If this force were
used to generate power instead of wasting it on the brakes,
determine the maximum amount of power that could ideally be
generated.
http://i.imgur.com/jb67dnd.jpg
(20%) (6) Glycerin at 40℃ with density ρ=1252kg/m^3 and dynamic viscosity
μ=0.27kg/ms is flowing through a 4-cm-diameter horizontal smooth
pipe with an average velocity of 3.5m/s. Determine the pressure
drop per 10m of the pipe and the required pumping power.
http://i.imgur.com/S2IZgsv.jpg

Links booklink

Contact Us: admin [ a t ] ucptt.com